QM M04 2

From QED

< PlasmaWiki | Prelims(Link to this page as [[PlasmaWiki/Prelims/QM M04 2]])
Jump to: navigation, search

First Solution

PlasmaWiki_Prelims_QM_M04_2.pdf (help ยท info)

Second Solution

The Hamiltonian is:

\mathcal{H}=a\vec{S}_{1}\cdot\vec{S}_{2}

Which we can rewrite using the dot product of the spin operator:

\mathcal{H}=\frac{a}{2}\left[J^{2}-S_{1}^{2}-S_{2}^{2}\right]

Particle 1 has spin S, particle 2 has spin 1/2, so we will get:

E=\frac{a}{2}\left[j(j+1)\hbar^{2}-S(S+1)\hbar^{2}-\frac{3\hbar^{2}}{4}\right]

The total spin j=S\pm1/2:

E=\frac{a\hbar^{2}}{2}\left[\left(S\pm\frac{1}{2}\right)\left(S\pm\frac{1}{2}+1\right)-S(S+1)-\frac{3}{4}\right]
E=\frac{a\hbar^{2}}{2}\left[S^{2}\pm\frac{S}{2}+S\pm\frac{S}{2}+\frac{1}{4}\pm\frac{1}{2}-S^{2}-S-\frac{3}{4}\right]=\frac{a\hbar^{2}}{2}\left[\pm S\pm\frac{1}{2}-\frac{1}{2}\right]

The energy S+\frac{1}{2} (thus taking on 2S + 2 states), while j=S-\frac{1}{2}, so it takes on 2S states.

Since we have two spin 1/2 particles, we can express every state as the up or down of each of the two particles:

\left|1,1\right\rangle =\left|++\right\rangle
\left|1,0\right\rangle =\frac{1}{\sqrt{2}}\left(\left|+-\right\rangle +\left|-+\right\rangle \right)
\left|1,-1\right\rangle =\left|--\right\rangle
\left|0,0\right\rangle =\frac{1}{\sqrt{2}}\left(\left|+-\right\rangle -\left|-+\right\rangle \right)

We apply the hamiltonian (and use our knowledge of Pauli Matrices to apply Sx and Sy to Sz eigenstates):

\mathcal{H}\left|1,1\right\rangle =a\left(S_{1}^{x}S_{2}^{x}+S_{1}^{y}S_{2}^{y}+S_{1}^{z}S_{2}^{z}\right)\left|1,1\right\rangle
\mathcal{H}\left|1,1\right\rangle =a\left(\frac{\hbar^{2}}{4}\left|--\right\rangle -\frac{\hbar^{2}}{4}\left|--\right\rangle +\frac{\hbar^{2}}{4}\left|++\right\rangle \right)=\frac{a\hbar^{2}}{4}\left|1,1\right\rangle

And:

\mathcal{H}\left|1,-1\right\rangle =a\left(\frac{\hbar^{2}}{4}\left|++\right\rangle -\frac{\hbar^{2}}{4}\left|++\right\rangle +\frac{\hbar^{2}}{4}\left|--\right\rangle \right)=\frac{a\hbar^{2}}{4}\left|1,-1\right\rangle

Next:

\mathcal{H}\left|1,0\right\rangle =\frac{a}{\sqrt{2}}\left(\frac{\hbar^{2}}{4}\left|-+\right\rangle +\frac{\hbar^{2}}{4}\left|+-\right\rangle +\frac{\hbar^{2}}{4}\left|-+\right\rangle +\frac{\hbar^{2}}{4}\left|+-\right\rangle -\frac{\hbar^{2}}{4}\left|+-\right\rangle -\frac{\hbar^{2}}{4}\left|-+\right\rangle \right)
\mathcal{H}\left|1,0\right\rangle =\frac{a\hbar^{2}}{4}\left|1,0\right\rangle

Lastly:

\mathcal{H}\left|0,0\right\rangle =\frac{a}{\sqrt{2}}\left(\frac{\hbar^{2}}{4}\left|-+\right\rangle -\frac{\hbar^{2}}{4}\left|+-\right\rangle +\frac{\hbar^{2}}{4}\left|-+\right\rangle -\frac{\hbar^{2}}{4}\left|+-\right\rangle -\frac{\hbar^{2}}{4}\left|+-\right\rangle +\frac{\hbar^{2}}{4}\left|-+\right\rangle \right)
\mathcal{H}\left|0,0\right\rangle =-\frac{3a\hbar^{2}}{4}\left|0,0\right\rangle

So the eigenvalue \left|1,-1\right\rangle , while the eigenvalue \left|0,0\right\rangle .

We use the same eigenvectors as they are all eigenvectors of Sz. Finding eigenvalues:

\mathcal{H}_{B}\left|1,1\right\rangle =\left(\frac{a\hbar^{2}}{4}+b\left(S_{1}^{z}-S_{2}^{z}\right)\right)\left|++\right\rangle =\left(\frac{a\hbar^{2}}{4}+b\left(\frac{\hbar}{2}-\frac{\hbar}{2}\right)\right)\left|++\right\rangle
\mathcal{H}_{B}\left|1,1\right\rangle =\frac{a\hbar^{2}}{4}\left|1,1\right\rangle

Similarly:

\mathcal{H}_{B}\left|1,-1\right\rangle =\left(\frac{a\hbar^{2}}{4}+b\left(-\frac{\hbar}{2}+\frac{\hbar}{2}\right)\right)\left|1,-1\right\rangle =\frac{a\hbar^{2}}{4}\left|1,-1\right\rangle

For \left|1,0\right\rangle :

\mathcal{H}_{B}\left|1,0\right\rangle =\frac{a\hbar^{2}}{4}\left|1,0\right\rangle +\frac{b}{\sqrt{2}}\left(\left(\frac{\hbar}{2}\left|+-\right\rangle -\frac{\hbar}{2}\left|-+\right\rangle \right)-\left(-\frac{\hbar}{2}\left|+-\right\rangle +\frac{\hbar}{2}\left|-+\right\rangle \right)\right)
\mathcal{H}_{B}\left|1,0\right\rangle =\frac{a\hbar^{2}}{4}\left|1,0\right\rangle +b\hbar\left|0,0\right\rangle

For \left|0,0\right\rangle :

\mathcal{H}_{B}\left|0,0\right\rangle =-\frac{3a\hbar^{2}}{4}\left|0,0\right\rangle +\frac{b}{\sqrt{2}}\left(\left(\frac{\hbar}{2}\left|+-\right\rangle +\frac{\hbar}{2}\left|-+\right\rangle \right)-\left(-\frac{\hbar}{2}\left|+-\right\rangle -\frac{\hbar}{2}\left|-+\right\rangle \right)\right)
\mathcal{H}_{B}\left|0,0\right\rangle =-\frac{3a\hbar^{2}}{4}\left|0,0\right\rangle +b\hbar\left|1,0\right\rangle

If we only consider these two states, then:

\mathcal{H}_{B}=\left(\begin{matrix} \frac{a\hbar^{2}}{4} &amp; b\hbar\\ b\hbar &amp; -\frac{3a\hbar^{2}}{4}\end{matrix}\right)

Which gives us the determinant:

\left|\begin{matrix} \lambda-\frac{a\hbar^{2}}{4} &amp; -b\hbar\\ -b\hbar &amp; \lambda+\frac{3a\hbar^{2}}{4}\end{matrix}\right|=0
\left(\lambda-\frac{a\hbar^{2}}{4}\right)\left(\lambda+\frac{3a\hbar^{2}}{4}\right)-b^{2}\hbar^{2}=0

Which has solutions \lambda_{-}=-a\hbar^{2}/4-\hbar\sqrt{b^{2}+a^{2}\hbar^{2}/4}. I will not find the normalized eigenstates because it is not fun.

This page was recovered in October 2009 from the Plasmagicians page on Prelim_M04_QM2 dated 14:05, 24 April 2006.

Personal tools